Hint needed for lipschitz problem.

  • Thread starter Thread starter JakobReed
  • Start date Start date
  • Tags Tags
    Lipschitz
Click For Summary
The discussion focuses on the Lipschitz condition as a sufficient criterion for uniqueness in solutions to differential equations. It highlights that a function f(x, y) must satisfy the Lipschitz condition to ensure a unique solution, specifically through the inequality |f(x,y1)−f(x,y2)|≤k|y1−y2| for some constant k. Participants are encouraged to explore an initial value problem (IVP) with two solutions where f(x, y) does not meet the Lipschitz condition, suggesting that continuous but non-differentiable functions may serve as examples. A hint is provided to investigate the differential equation dy/dx = f(x,y) with f being continuous but not differentiable, potentially using fractional powers of y. The discussion emphasizes the importance of understanding the implications of the Lipschitz condition in the context of differential equations.
JakobReed
Messages
1
Reaction score
0
a sufficient condition for uniqueness is the Lipschitz condition:

On a domain D of the plane, the function f (x, y) is said to satisfy the Lipschitz condition for a constant k > 0 if:

|f(x,y1)−f(x,y2)|≤k|y1−y2|

for all points (x,y1) and (x,y2) in D.

Give an example of an IVP with two solutions on a domain (say, a rectangle) and show that the function f(x,y) appearing in the differential equation fails to be Lipschitz for any k > 0.

i really have no idea where to begin. Can i get a hint or a suggestion from someone?
 
Physics news on Phys.org
"Lipschitz" is intermediate between "continuous" and "differentiable" in strength (any Lipschitz function is continuous but not vice versa; any differentiable function is Lipschitz but not vice versa). Start by looking at dy/dx= f(x,y) where f is continuous but not differentiable with respect to y. Try fractional powers of y.
 
Question: A clock's minute hand has length 4 and its hour hand has length 3. What is the distance between the tips at the moment when it is increasing most rapidly?(Putnam Exam Question) Answer: Making assumption that both the hands moves at constant angular velocities, the answer is ## \sqrt{7} .## But don't you think this assumption is somewhat doubtful and wrong?

Similar threads

  • · Replies 3 ·
Replies
3
Views
2K
  • · Replies 1 ·
Replies
1
Views
2K
Replies
3
Views
2K
  • · Replies 9 ·
Replies
9
Views
4K
  • · Replies 5 ·
Replies
5
Views
2K
  • · Replies 3 ·
Replies
3
Views
7K
  • · Replies 3 ·
Replies
3
Views
2K
  • · Replies 2 ·
Replies
2
Views
3K
  • · Replies 12 ·
Replies
12
Views
4K
  • · Replies 28 ·
Replies
28
Views
4K